Thursday, February 2, 2017

Case Study Of A 20-year-old woman with no significant past medical history presents with a 2-month history of episodic shortness of breath.



A 20-year-old woman with no significant past medical history presents with a 2-month history of episodic shortness of breath. These symptoms began with an upper respiratory tract infection. She has fits of coughing and trouble catching her breath with exertion. She states that her breath “sounds like whistles” at times. She tried a friend’s albuterol inhaler with some improvement and wonders if she has asthma. On exam, she is breathing comfortably at 16 times per minute and her oxygen saturation is 96% on room air. Her lungs are clear to auscultation, and the remainder of her exam is unremarkable. You want to better categorize, this patient’s disease.

Which of the following tests is most appropriate to order now?
A) Spirometry.
B) Chest x-ray.
C) Arterial blood gas (ABG).
D) Methacholine challenge.
E) Chest CT.

Answer And Discussion
The correct answer is “A.” Spirometry.

Since this patient has symptoms of bronchospasm, spirometry will be essential in determining if there is objective evidence of obstructive lung disease. However, spirometry results are often normal in mild cases of asthma, especially when the patient is asymptomatic. Bronchoprovocation testing, with methacholine or histamine, may be useful in such cases, but should follow basic spirometry. Although chest radiography (x-ray or CT) may reveal an unsuspected process, it is not indicated in
otherwise healthy patients with symptoms of bronchospasm.
Bacterial pneumonia is a potential precipitant of bronchospasm that may be diagnosed on chest x-ray, but this patient has no constitutional symptoms (like fever) associated with serious bacterial infection. ABG levels may be helpful when a patient presents with respiratory distress but certainly not in the office setting.

If this patient has mild asthma, which of the following pulmonary function test results would you expect to find?
A) Forced vital capacity (FVC) 50% of predicted.
B) Forced expiratory volume in 1 second (FEV1)
100% of predicted.
C) FEV1/FVC ratio <0.7.
D) Total lung capacity (TLC) 50% of predicted.
E) FEV1/TLC <0.7.

Answer And Discussion

The correct answer is “C.” FEV1/FVC ratio <0.7.

Patients with asthma will have a decreased FEV1. The FVC may fall as well, but FEV1 falls first and to a greater degree as the lung becomes obstructed. The ratio of FEV1/FVC is very sensitive to airflow limitations, and FEV1/FVC <0.7 (not predicted, just the ratio of the two numbers) is generally considered diagnostic of obstructive airway disease.
The rest are incorrect. TLC is not measured by spirometry (which is why “E” is incorrect).

Your patient’s office spirometry shows the following:
Normal FVC.
FEV1 82% predicted.
FEV1/FVC 0.68.

These findings are most consistent with which of the following?
A) Normal spirometry.
B) Obstructive lung disease.
C) End-stage emphysema.
D) Interstitial fibrosis.

Answer And Discussion
The correct answer is “B.”  Obstructive lung disease.

Always go first to the FEV1/FVC ratio. In this case, it is <0.70, which is suggestive of airway obstruction. The information provided here lacks data regarding reversibility, so you could not really differentiate between chronic obstructive pulmonary disease (COPD) and asthma. But this is clearly not end-stage emphysema, so “C” is incorrect.
“D” is incorrect. Interstitial fibrosis is generally marked by a restrictive pattern on spirometry and decreased TLC. Both flow rate (e.g., FEV1) and FVC are decreased in interstitial lung diseases but in proportion to each other. Thus, the FEV1/FVC is often normal or elevated.

Six months after you discuss her findings and prescribe inhaled beta-agonist therapy, she returns with complaints of continued wheezing and difficulty breathing. Her symptoms are brought on by cold weather and exercise and she uses her inhaler two times per week. She woke up two nights over the last 6 months with shortness of breath and coughing. Her albuterol still works for these symptoms, but she finds them bothersome and asks, “Why haven’t I gotten over this?”

How would you categorize this patient’s respiratory state?
A) Intermittent asthma.
B) Mild persistent asthma.
C) Moderate persistent asthma.
D) Severe persistent asthma.
E) Recurrent lower respiratory tract infections.

Answer And Discussion
The correct answer is “A.” Intermittent asthma.

According to the National
Asthma Education and Prevention Program (2007 NHLBI/NAEPPguidelines), your patient meets the criteria for intermittent asthma. In such patients, mild symptoms correspond to an FEV1 (not an FEV1/FVC ratio) that is greater than 80% predicted.

Which of the following is most appropriate for this patient given that she has intermittent asthma?
A) Add theophylline.
B) Add montelukast.
C) Continue albuterol as needed.
D) Schedule albuterol every 4 hours.
E) Prednisone 5 mg daily.

Answer And Discussion
The correct answer is “C.” Continue albuterol as needed.
 As already discussed, this patient appears to have intermittent asthma. She is in no respiratory distress, is oxygenating normally, and is still responding well to albuterol by her report. Although there is some debate about the role of inhaled steroids in intermittent asthma, the NAEPP and most experts do not recommend their use. Oral prednisone is certainly not indicated in this case. She should be continued on a short-acting inhaled beta- 2 agonist, such as albuterol, without the addition of another medication.
“D” is incorrect. Scheduled albuterol actually yields less effective symptom control than does PRN use.

Your patient goes on to develop more frequent recurrent symptoms, such that she is using her albuterol inhaler more than three times per week, although her nighttime symptoms are rare.

Which medication is the most appropriate next step in treating this patient’s asthma?
A) Inhaled triamcinolone.
B) Inhaled salmeterol.
C) Inhaled cromolyn sodium.
D) Inhaled ipratropium.
E) Oral montelukast.

Answer And Discussion
The correct answer is “A.”  Inhaled triamcinolone.

Your patient now has mild persistent asthma and should be started on an inhaled steroid. When asthma symptoms become more persistent (i.e., when they occur >2 days per week or the patient awakens from sleep >2 times per month), the inflammatory component of the disease should be addressed while simultaneously treating the bronchospastic component with a short-acting beta-2 agonists. Anti-inflammatory drugs are the mainstay of chronic asthma therapy, and inhaled corticosteroids are the most efficacious with the fewest side effects.
Although ipratropium, cromolyn sodium, and montelukast have a place in asthma treatment, none of
these medications is a first-line agent. Ipratropium works through its bronchodilatory effects, while cromolyn sodium is amast cell stabilizer. Montelukast is a leukotriene inhibitor. The long-acting inhaled beta-2 agonists, such as salmeterol, are only recommended at Steps 3 and higher of persistent asthma control.

Your patient does quite well over the next year, having very few exacerbations. During one of her visits, you note slightly edematous nasal mucosa and nasal polyps. You prescribe intranasal steroids.

One night when you are on call, she comes in severely dyspneic with audible wheezing. She talks in
two- or three-word phrases and reports headache today, which she treated with aspirin (something she
never takes but a friend gave her thinking it was acetaminophen). Her asthma attack started about an
hour after the aspirin dose. She has been otherwise well. She denies fever, rhinorrhea, nasal congestion, and sore throat. Her respiratory rate is 40, heart rate 120, and oxygen saturation 88% on room air. She has poor air movement on auscultation of her lung fields.

Which of the following is the most likely reason for this patient’s acute exacerbation of asthma?
A) Viral upper respiratory infection (URI).
B) Sinusitis.
C) Noncompliance with inhaled albuterol.
D) Sensitivity to aspirin.
E) Noncompliance with nasal steroids.

Answer And Discussion
The correct answer is “D.” Sensitivity to aspirin.

It is likely that this patient has aspirin sensitivity. Up to 10% of adults with asthma have the clinical triad of asthma, aspirin sensitivity, and nasal polyposis. Patients with asthma should be warned about the potential for exacerbations resulting from consumption of aspirin and nonsteroidal anti-inflammatory drugs (NSAIDs).
The drug-induced bronchial constriction caused by these medications can have an abrupt onset with severe symptoms. Patients with aspirin sensitivity can be desensitized with daily administration of small amounts of aspirin, but this should be done carefully with close supervision.
Although viral URIs frequently cause exacerbations of asthma, your patient did not report antecedent
symptoms of such an infection.

After a brief hospitalization, your patient recovers nicely. Prior to this incident involving aspirin, she had been free of exacerbations for about a month 

In addition to a short course of oral steroids, which of the following medication regimens do you prescribe for this patient with aspirinsensitive asthma at discharge?
A) Inhaled triamcinolone and inhaled albuterol as a“rescue.”
B) Inhaled triamcinolone, oral montelukast, and inhaled albuterol as a “rescue.”
C) Oral montelukast and inhaled albuterol as a “rescue.”
D) Inhaled albuterol as a “rescue.”
E) Inhaled salmeterol and inhaled triamcinolone.

Answer And Discussion
The correct answer is “B.”  Inhaled triamcinolone, oral montelukast, and inhaled albuterol as a “rescue.”

Leukotriene inhibitors (e.g., montelukast, zafirlukast) do not have a primary role in asthma management. However, these medications have demonstrated effectiveness in reducing symptoms and improving peak flow in patients with aspirin-sensitive asthma.
Leukotriene inhibitors should be used only in asthma patients who are already using a corticosteroid inhaler—or those who cannot tolerate inhaled corticosteroid therapy. Therefore, “C” is not an appropriate choice. “D” is incorrect because there is no anti-inflammatory. Although “E” offers an antiinflammatory agent, there is no rescue inhaler, and patients with asthma must always have access to a short-acting inhaled bronchodilator.

Which of the following medications, when used alone as maintenance therapy in persistent asthma, is associated with an increased riskof asthma-related mortality?
A) Inhaled fluticasone.
B) Inhaled salmeterol.
C) Oral zafirlukast.
D) Oral prednisone.

Answer And Discussion
The correct answer is “B.” Inhaled salmeterol.

 Inhaled salmeterol, when used alone as a controller agent for asthma, has been associated with a two- to fourfold increase in the risk of death related to asthma or other respiratory conditions. Thus, the Food and Drug Administration (FDA) has mandated a “black box” warning be applied to salmeterol-containing products.

Objectives Learned In This Case Study: 

  • To  Identify the  triggers of bronchospasm
  • Evaluate symptoms of wheezing and dyspnea
  • Classify asthma
  • Prescribe appropriate medications for intermittent and mild persistent asthma
  • Describe the triad of asthma, aspirin sensitivity, and nasal polyposis

1 comment: